Difference between revisions of "2017 AMC 8 Problems/Problem 25"

(Problem 25)
(Solution)
Line 10: Line 10:
  
 
Let the centers of the circles containing arcs <math>\overarc{SR}</math> and <math>\overarc{TR}</math> be <math>S'</math> and <math>T'</math>, respectively. Extend <math>\overline{US}</math> and <math>\overline{UT}</math> to <math>S'</math> and <math>T'</math>. The area of the figure is equal to the area of equilateral triangle <math>\triangle US'T'</math> minus the combined area of the <math>2</math> sectors of the circles. The area of <math>\triangle US'T'</math> is <math>\frac{\sqrt 3}{4} \cdot 4^2 = 4\sqrt 3.</math> The combined area of the <math>2</math> sectors is <math>2 \cdot \frac 16 \cdot \pi \cdot 2^2 = \frac{4\pi}{3}.</math> Our final answer is then <math>\boxed{\textbf{(B)}\ 4\sqrt{3}-\frac{4\pi}{3}}.</math>
 
Let the centers of the circles containing arcs <math>\overarc{SR}</math> and <math>\overarc{TR}</math> be <math>S'</math> and <math>T'</math>, respectively. Extend <math>\overline{US}</math> and <math>\overline{UT}</math> to <math>S'</math> and <math>T'</math>. The area of the figure is equal to the area of equilateral triangle <math>\triangle US'T'</math> minus the combined area of the <math>2</math> sectors of the circles. The area of <math>\triangle US'T'</math> is <math>\frac{\sqrt 3}{4} \cdot 4^2 = 4\sqrt 3.</math> The combined area of the <math>2</math> sectors is <math>2 \cdot \frac 16 \cdot \pi \cdot 2^2 = \frac{4\pi}{3}.</math> Our final answer is then <math>\boxed{\textbf{(B)}\ 4\sqrt{3}-\frac{4\pi}{3}}.</math>
 +
 +
<asy>draw((1,1.732)--(2,3.464)--(3,1.732));
 +
draw((1,1.732)--(0,0)--(4,0)--(3,1.732));
 +
draw(arc((0,0),(2,0),(1,1.732)));
 +
draw(arc((4,0),(3,1.732),(2,0)));
 +
label("$U$", (2,3.464), N);
 +
label("$S$", (1,1.732), W);
 +
label("$T$", (3,1.732), E);
 +
label("$R$", (2,0), S);
 +
label("$S'$", (0,0), W);
 +
label("$T'$", (4,0), E);</asy>
  
 
~nukelauncher
 
~nukelauncher

Revision as of 14:56, 22 November 2017

Problem 25

In the figure shown, $\overline{US}$ and $\overline{UT}$ are line segments each of length 2, and $m\angle TUS = 60^\circ$. Arcs $\overarc{TR}$ and $\overarc{SR}$ are each one-sixth of a circle with radius 2. What is the area of the region shown?

[asy]draw((1,1.732)--(2,3.464)--(3,1.732)); draw(arc((0,0),(2,0),(1,1.732))); draw(arc((4,0),(3,1.732),(2,0))); label("$U$", (2,3.464), N); label("$S$", (1,1.732), W); label("$T$", (3,1.732), E); label("$R$", (2,0), S);[/asy]

$\textbf{(A) }3\sqrt{3}-\pi\qquad\textbf{(B) }4\sqrt{3}-\frac{4\pi}{3}\qquad\textbf{(C) }2\sqrt{3}\qquad\textbf{(D) }4\sqrt{3}-\frac{2\pi}{3}\qquad\textbf{(E) }4+\frac{4\pi}{3}$

Solution

Let the centers of the circles containing arcs $\overarc{SR}$ and $\overarc{TR}$ be $S'$ and $T'$, respectively. Extend $\overline{US}$ and $\overline{UT}$ to $S'$ and $T'$. The area of the figure is equal to the area of equilateral triangle $\triangle US'T'$ minus the combined area of the $2$ sectors of the circles. The area of $\triangle US'T'$ is $\frac{\sqrt 3}{4} \cdot 4^2 = 4\sqrt 3.$ The combined area of the $2$ sectors is $2 \cdot \frac 16 \cdot \pi \cdot 2^2 = \frac{4\pi}{3}.$ Our final answer is then $\boxed{\textbf{(B)}\ 4\sqrt{3}-\frac{4\pi}{3}}.$

[asy]draw((1,1.732)--(2,3.464)--(3,1.732)); draw((1,1.732)--(0,0)--(4,0)--(3,1.732)); draw(arc((0,0),(2,0),(1,1.732))); draw(arc((4,0),(3,1.732),(2,0))); label("$U$", (2,3.464), N); label("$S$", (1,1.732), W); label("$T$", (3,1.732), E); label("$R$", (2,0), S); label("$S'$", (0,0), W); label("$T'$", (4,0), E);[/asy]

~nukelauncher